Difference between revisions of "2004 AMC 10A Problems/Problem 18"

m (See Also)
(redirection)
 
(One intermediate revision by one other user not shown)
Line 1: Line 1:
==Problem==
+
#REDIRECT [[2004 AMC 12A Problems/Problem 14]]
A sequence of three real numbers forms an arithmetic progression with a first term of 9.  If 2 is added to the secon term and 20 is added to the third term, the three resulting numbers form a geometric progression.  What is the smallest possible value for the third term of the geometric progression?
 
 
 
<math> \mathrm{(A) \ } 1 \qquad \mathrm{(B) \ } 4 \qquad \mathrm{(C) \ } 36 \qquad \mathrm{(D) \ } 49 \qquad \mathrm{(E) \ } 81  </math>
 
 
 
==Solution==
 
Let d be the difference between terms in the arithmetic progression, such that first three terms are 9, 9+d, and 9+2d.  The terms of the geometric progression will be 9, 11+d, and 29+2d.  Because they are in a geometric progression, we can say
 
 
 
<math>(11+d)^2=9(29+2d)</math>, note that the terms of a geometric progression would be <math>\frac{x}r</math>, x, and <math>xr</math>, so <math>x^2=\frac{x}r\times r</math>.
 
 
 
Solve this equation.
 
 
 
<math>d^2+4d-140=0</math>
 
 
 
<math>(d-10)(d+14)=0</math>
 
 
 
Substituting <math>d=10</math> into the geometric progression gives the terms 9, 21, and 49; substituting <math>d=14</math> gives 9, -3, and 1.  Therefore, the smallest possible 3rd term is 1 <math>\Rightarrow\mathrm{(A)}</math>.
 
 
 
== See also ==
 
{{AMC10 box|year=2004|ab=A|num-b=17|num-a=19}}
 
 
 
[[Category:Introductory Algebra Problems]]
 

Latest revision as of 16:15, 5 December 2007